www.vorhilfe.de
Vorhilfe

Kostenlose Kommunikationsplattform für gegenseitige Hilfestellungen.
Hallo Gast!einloggen | registrieren ]
Startseite · Forum · Wissen · Kurse · Mitglieder · Team · Impressum
Forenbaum
^ Forenbaum
Status Englisch
  Status Grammatik
  Status Lektüre
  Status Korrekturlesen
  Status Übersetzung
  Status Sonstiges (Englisch)

Gezeigt werden alle Foren bis zur Tiefe 2

Navigation
 Startseite...
 Neuerdings beta neu
 Forum...
 vorwissen...
 vorkurse...
 Werkzeuge...
 Nachhilfevermittlung beta...
 Online-Spiele beta
 Suchen
 Verein...
 Impressum
Das Projekt
Server und Internetanbindung werden durch Spenden finanziert.
Organisiert wird das Projekt von unserem Koordinatorenteam.
Hunderte Mitglieder helfen ehrenamtlich in unseren moderierten Foren.
Anbieter der Seite ist der gemeinnützige Verein "Vorhilfe.de e.V.".
Partnerseiten
Weitere Fächer:

Open Source FunktionenplotterFunkyPlot: Kostenloser und quelloffener Funktionenplotter für Linux und andere Betriebssysteme
Forum "Uni-Stochastik" - Poissonverteilung
Poissonverteilung < Stochastik < Hochschule < Mathe < Vorhilfe
Ansicht: [ geschachtelt ] | ^ Forum "Uni-Stochastik"  | ^^ Alle Foren  | ^ Forenbaum  | Materialien

Poissonverteilung: Frage (beantwortet)
Status: (Frage) beantwortet Status 
Datum: 20:25 Di 31.01.2006
Autor: sternchen19.8

Aufgabe
Betrachtet wird die Poissonverteilung zum Parameter [mm] \lambda. [/mm] Es sei bekannt, dass die Wahrscheinlichkeit der Menge {3} doppelt so groß ist, wie die Wahrscheinlichkeit der Menge {7}. Wie groß ist [mm] \lambda? [/mm]

Bin gerade bei den Klausurvorbereitungen und habe die Aufgabe in mienen Aufzeichnungen gefunden.
Leider war aber keine Lösung dahinter. Da ich es für eine gute Übungsaufgabe halte, aber keine Idee habe, wie man es lösen kann, wollte ich euch fragen.
Hoffe es kann mir einer dabei helfen!

        
Bezug
Poissonverteilung: Antwort
Status: (Antwort) fertig Status 
Datum: 21:52 Di 31.01.2006
Autor: Stefan

Hallo sternchen!

Es soll

$P(X=3) = 2 [mm] \cdot [/mm] P(X=5)$,

also:

[mm] $e^{-\lambda} \cdot \frac{\lambda^3}{3!} [/mm] = 2 [mm] e^{-\lambda} \cdot \frac{\lambda^5}{5!}$ [/mm]

gelten.

Das wirst du sicherlich nach [mm] $\lambda$ [/mm] auflösen können, oder?

Liebe Grüße
Stefan

Bezug
Ansicht: [ geschachtelt ] | ^ Forum "Uni-Stochastik"  | ^^ Alle Foren  | ^ Forenbaum  | Materialien


^ Seitenanfang ^
www.englischraum.de
[ Startseite | Forum | Wissen | Kurse | Mitglieder | Team | Impressum ]